Diễn Đàn MathScopeDiễn Đàn MathScope
  Diễn Đàn MathScope
Ghi Danh Hỏi/Ðáp Thành Viên Social Groups Lịch Ðánh Dấu Ðã Ðọc

Go Back   Diễn Đàn MathScope > Sơ Cấp > Việt Nam và IMO > 2011

News & Announcements

Ngoài một số quy định đã được nêu trong phần Quy định của Ghi Danh , mọi người tranh thủ bỏ ra 5 phút để đọc thêm một số Quy định sau để khỏi bị treo nick ở MathScope nhé !

* Nội quy MathScope.Org

* Một số quy định chung !

* Quy định về việc viết bài trong diễn đàn MathScope

* Nếu bạn muốn gia nhập đội ngũ BQT thì vui lòng tham gia tại đây

* Những câu hỏi thường gặp

* Về việc viết bài trong Box Đại học và Sau đại học


Trả lời Gởi Ðề Tài Mới
 
Ðiều Chỉnh Xếp Bài
Old 08-04-2011, 05:56 PM   #1
n.v.thanh
Moderator
 
n.v.thanh's Avatar
 
Tham gia ngày: Nov 2009
Bài gởi: 2,849
Thanks: 2,980
Thanked 2,537 Times in 1,008 Posts
Việt Nam Team Selection Test 2011-Đề thi, Đáp án và Danh sách Đội tuyển

Topic về kì thi chọn đội tuyển Việt Nam tham dự kì thi IMO 2011

-----------------------------------------------------

Đến hẹn lại lên,sau kì thi VMO 2011 diễn ra cách đây gần 2 tháng thì vào ngày mai và ngày kia Bộ GDĐT sẽ tổ chức kì thi chọn đội tuyển VN tham dự kì thi Olympic [Only registered and activated users can see links. ] diễn ra ở Hà Lan .
Nvthanh lại một lần nữa lập ra topic này nhằm tạo một nơi thảo luận có khoa học về kì thi này. Về cơ bản kì thi sẽ diễn ra vào hai ngày 8 và mùng 9 tháng 4 năm 2011(tức ngày mai và ngày kia) với sự góp mặt của 42 thí sinh đến từ khắp các vùng miền hải đảo của Tổ quốc Việt Nam yêu dấu.
Về công việc ngày mai thì do đội ngũ Mod của forum không ai tham gia kì thi quan trọng này nên vấn đề đề đóm có lẽ cứ mạnh ai nấy viết rồi các Mod sẽ edit thành một đề hoàn chỉnh vậy.

Vậy nhé, đã quán xuyến công việc xong. Cho Nvthanh gửi lời chúc tới các thí sinh, đặc biệt là các thí sinh đến từ Chuyên KHTN, cố giật vài vé đi Amsterdam nhé. Mọi người có thể chúc tụng thoải mái nhưng sáng mai T sẽ del để sao cho đề bài sẽ là ở Post 2, mong mọi người thông cảm.


[RIGHT][I][B]Nguồn: MathScope.ORG[/B][/I][/RIGHT]
 

thay đổi nội dung bởi: n.v.thanh, 08-04-2011 lúc 08:28 PM
n.v.thanh is offline   Trả Lời Với Trích Dẫn
The Following 9 Users Say Thank You to n.v.thanh For This Useful Post:
4eyes_l0vely (09-04-2011), AnhIsGod (15-04-2012), avip (08-04-2011), babylong (08-04-2011), long_chau2010 (13-04-2011), ltdung_t2k19 (20-04-2011), luatdhv (08-04-2011), Nguyenhuyen_AG (08-04-2011), ohmymath (21-04-2011)
Old 09-04-2011, 12:00 PM   #2
n.v.thanh
Moderator
 
n.v.thanh's Avatar
 
Tham gia ngày: Nov 2009
Bài gởi: 2,849
Thanks: 2,980
Thanked 2,537 Times in 1,008 Posts
Đề thi chọn đội tuyển Việt Nam tham dự kì thi Olympic Toán quốc tế 2011


Ngày thi thứ nhất 9/04/2011
Thời gian làm bài 240 phút


Bài 1(5 điểm)
Tại điểm (1;1) của mặt phẳng tọa độ Oxy có một con cào cào.Từ điểm đó,con cào cào chỉ nhảy đến các điểm nguyên dương khác theo quy tắc: từ điểm nguyên dương A,con cào cào nhảy đến điểm nguyên dương B nếu tam giác AOB có diện tích bằng $\dfrac{1}{2} $.
1/Tìm tất cả các điểm nguyên dương (m;n) mà con cào cào có thể nhảy đến sau một số hữu hạn bước,xuất phát từ điểm (1;1).
2/Giả sử (m;n) là một điểm nguyên dương có tính chất đã nêu ở câu 1/.Chứng minh rằng tồn tại một cách nhảy của con cào cào từ điểm (1;1) đến điểm (m;n) mà số bước nhảy không vượt quá |m-n|.
(Điểm (x;y) gọi là điểm nguyên dương nếu x và y là các số nguyên dương).

Bài 2(7,0 điểm)
Trên mặt phẳng cho (O ) và một điểm A nằm ngoài đường tròn đó.Qua A kẻ các tiếp tuyến tới (O),gọi B,C là tiếp điểm.Xét một điểm Pdi động trên tia đối của tia BA,Q là điểm di động trên tia đối của tia CA sao cho đường thẳng PQ tiếp xúc với (O).Qua P kẻ đường thằng song song với AC,cắt BC tại E.Qua Q kẻ đường thẳng song song với AB cắt BC tại F.Chứng minh rằng
1/Đường thẳng EQ luôn đi qua một điểm cố định M và FP luôn đi qua một điểm cố định N.
2/Tích PM.QN không đổi.

Bài 3(8 điểm)
Cho số nguyên $n\geq 3 $.Xét $n $ số thực $x_1,x_2,\ldots,x_n $ thỏa mãn đồng thời các điều kiện sau:

i/ $x_1\geq x_2\geq x_2\geq \ldots \geq x_n $
ii/ $x_1+x_2+\ldots+x_n=0 $
iii/$x_1^2+x_2^2+\ldots+x_n^2=n(n-1) $

Tìm giá trị lớn nhất và giá trị nhỏ nhất của tổng $S=x_1+x_2 $
[RIGHT][I][B]Nguồn: MathScope.ORG[/B][/I][/RIGHT]
 

thay đổi nội dung bởi: n.v.thanh, 11-04-2011 lúc 04:24 PM
n.v.thanh is offline   Trả Lời Với Trích Dẫn
The Following 13 Users Say Thank You to n.v.thanh For This Useful Post:
babylong (09-04-2011), cvppro (11-04-2011), duycvp (09-04-2011), hgly1996 (28-04-2011), horakhti1995 (10-04-2011), huynhcongbang (09-04-2011), Messi_ndt (09-04-2011), mnnn (07-05-2011), Nguyenhuyen_AG (09-04-2011), ohmymath (21-04-2011), shinomoriaoshi (09-04-2011), thaygiaocht (24-04-2015), winwave (09-04-2011)
Old 09-04-2011, 12:18 PM   #3
n.v.thanh
Moderator
 
n.v.thanh's Avatar
 
Tham gia ngày: Nov 2009
Bài gởi: 2,849
Thanks: 2,980
Thanked 2,537 Times in 1,008 Posts
Ngày thi thứ hai 10/4/2011
Thời gian làm bài 240 phút
Bài 4 (6,0 điểm)
Cho dãy ${a_n} $ thỏa mãn $a_0=1,a_1=3 $ và $a_{n+2}=1+\left \lfloor \frac{a^2_{n+1}}{a_n} \right \rfloor $ với mọi $n\geq0 $
Chứng minh rằng
$a_n.a_{n+2}-a^2_{n+1}=2^n $ với mọi số tự nhiên $n $

Bài 5(7,0 điểm)

Tìm tất cả các số nguyên dương $n $ sao cho $A=2^{n+2}.(2^n-1)-8.3^n+1 $ là số chính phương.

Bài 6(7,0 điểm)
Cho n là một số nguyên lớn hơn 1.Có n học sinh ngồi quanh một chiếc bàn tròn,mỗi em có một số kẹo (có thể có em không có một chiếc kẹo nào) và tổng số kẹo của tất cả các em là một bội của n.Các em thực hiện việc chuyển kẹo như sau:
Với số kẹo mỗi em có lúc đầu,có ít nhất một em có nhiều kẹo hơn bạn ngồi bên phải mình thì một em (tùy ý) trong những em như thế chuyển một chiếc kẹo của mình cho bạn ngồi ngay bên phải.Với số kẹo mỗi em có sau lần chuyển thứ nhất,nếu có ít nhất một em có nhiều kẹo hơn bạn ngồi bên phải thì một em (tùy ý) trong số những em như thế lại tiếp tục chuyển 1 chiếc lẹo của mình cho bạn ngồi bên phải.Quá trình chuyển kẹo cứ thế được tiếp tục.
Chứng minh rằng sau một số hữu hạn lần chuyển kẹo như vậy,tất cả các em đều có số kẹo như nhau.
Hết

[RIGHT][I][B]Nguồn: MathScope.ORG[/B][/I][/RIGHT]
 
File Kèm Theo
Kiểu File : pdf VietnamTST2011.pdf (42.3 KB, 347 lần tải)

thay đổi nội dung bởi: novae, 11-04-2011 lúc 10:59 PM
n.v.thanh is offline   Trả Lời Với Trích Dẫn
The Following 6 Users Say Thank You to n.v.thanh For This Useful Post:
cattuong (30-05-2011), hoanghai_vovn (10-04-2011), horakhti1995 (10-04-2011), Nguyenhuyen_AG (10-04-2011), ohmymath (21-04-2011), vthiep94 (16-04-2011)
Old 09-04-2011, 03:45 PM   #4
huynhcongbang
Administrator

 
huynhcongbang's Avatar
 
Tham gia ngày: Feb 2009
Đến từ: Ho Chi Minh City
Bài gởi: 2,413
Thanks: 2,165
Thanked 4,188 Times in 1,381 Posts
Gửi tin nhắn qua Yahoo chát tới huynhcongbang
Mình ủng hộ bài 2 trước:
Trước hết, ta chứng minh bổ đề sau:
* Cho tam giác ABC ngoại tiếp (I) có tiếp điểm của (I) lên AB, AC lần lượt là E, F. Đường thẳng qua B, song song với AC cắt EF tại K; CK cắt AB tại G. Chứng minh rằng tam giác AGI vuông tại I.

Chứng minh:
Do BK // AC nên tam giác BKF cân tại B, suy ra: $BK=BF = p-b $.



Theo định lí Thales thì:
$\frac{BG}{AG}=\frac{BK}{AC}=\frac{p-b}{b} \Rightarrow \frac{AB}{AG} = \frac{p}{b}\Rightarrow AG = \frac{bc}{p} $
Mà $AF=p-a $ nên $\frac{AF}{AG}=\frac{p(p-a)}{bc} $.
Ta cũng có: $AI = \frac{AF}{\sin \frac{A}{2}}, AH = AF. \sin \frac{A}{2} $.
Do đó: $\frac{AH}{AI}=\sin^2 \frac{A}{2} = \frac{1-\cos A}{2} = \frac{1-\dfrac{b^2+c^2-a^2}{2bc}}{2} = \frac{p(p-a)}{bc} $
Suy ra: $\frac{AF}{AG} =\frac{AH}{AI} $.
Tức là AGI vuông tại I.
Bổ đề được chứng minh.

Trở lại bài toán:



1/ Gọi M, N lần lượt là giao điểm của QE với AB và PF với AC.
Theo bổ đề trên, ta thấy rằng tam giác OMA và ONA lần lượt vuông tại O nên các điểm M, N cố định.
2/ Đặt $AB=AC=a, BP=x, CQ=y $. Chu vi của tam giác APQ là $2(a+x+y) $.
Theo bổ đề trên, ta tính được:
$PM =AP - \frac{2AP.PQ}{AP+AQ+PQ}=\frac{(a+x)x}{a+x+y} $ và
$QN =AQ - \frac{2AQ.PQ}{AP+AQ+PQ}=\frac{(a+y)y}{a+x+y} $.
Ta sẽ chứng minh rằng $\frac{xy(a+x)(a+y)}{(a+x+y)^2} $ không đổi.
Thật vậy:
Diện tích của tam giác APQ cùng bằng:
$R(AP+AQ+PQ) = \sin \widehat{BAC}.AP.AQ \Leftrightarrow \frac{(a+x)(a+y)}{a+x+y}=\frac{R}{ \sin \widehat{BAC}} $.
Tức là tỉ số: $\frac{(a+x)(a+y)}{a+x+y} = k $ không đổi, với $k=\frac{R}{ \sin \widehat{BAC}} $
Từ $(a+x)(a+y)=k(a+x+y) \Leftrightarrow a(a+x+y)+xy = k(a+x+y) \Leftrightarrow a+ \frac{xy}{a+x+y} = k $, suy ra tỉ số $\frac{xy}{a+x+y} $ cũng không đổi.
Ta có đpcm.
[RIGHT][I][B]Nguồn: MathScope.ORG[/B][/I][/RIGHT]
 

thay đổi nội dung bởi: huynhcongbang, 09-04-2011 lúc 04:10 PM
huynhcongbang is offline   Trả Lời Với Trích Dẫn
The Following 4 Users Say Thank You to huynhcongbang For This Useful Post:
babylong (09-04-2011), buikhacduong (09-04-2011), n.v.thanh (09-04-2011), nhox12764 (09-04-2011)
Old 09-04-2011, 06:28 PM   #5
shido_soichua
Maths is my life
 
shido_soichua's Avatar
 
Tham gia ngày: Oct 2009
Đến từ: Ninh Bình
Bài gởi: 300
Thanks: 31
Thanked 132 Times in 76 Posts
Gửi tin nhắn qua Yahoo chát tới shido_soichua
Thử bài 3 phát
Do $x_1^2+x_2^2+\ldots+x_n^2=n(n-1) $ nên $x_1^2+x_2^2=n(n-1)-x_2^2\ldots+x_n^2 $
Lại có
$(n-1)(x_3^2+x_4^2+.......+x_n^2)\geq (x_3+x_4+.....+x_n)^{2}= (x_1+x_2)^{2} $
$(x_1^2+x_2^2)\geq (x_1+x_2)^2 $
nên suy ra $n(x_1+x_2)^2\leq 2n(n-1)(n-2) $
Từ đấy suy ra $Max f=\sqrt{2(n-1)(n-2)} $ khi $x_1=x_2=\sqrt{\frac{(n-1)(n-2)}{2}} $ và $x_3=x_4=......x_n=-\sqrt{2(n-1} $
$min f=-\sqrt{2(n-1)(n-2)} $ khi $x_1=x_2=-\sqrt{\frac{(n-1)(n-2)}{2}} $ và $x_3=x_4=......x_n=\sqrt{2(n-1} $
Ôi min sai mất rùi
[RIGHT][I][B]Nguồn: MathScope.ORG[/B][/I][/RIGHT]
 
__________________
http://luongvantuy.org/forum.php
Chuyên Văn - Lương Văn Tụy

thay đổi nội dung bởi: shido_soichua, 09-04-2011 lúc 06:37 PM
shido_soichua is offline   Trả Lời Với Trích Dẫn
The Following 2 Users Say Thank You to shido_soichua For This Useful Post:
4eyes_l0vely (09-04-2011), buikhacduong (11-04-2011)
Old 09-04-2011, 10:01 PM   #6
huynhcongbang
Administrator

 
huynhcongbang's Avatar
 
Tham gia ngày: Feb 2009
Đến từ: Ho Chi Minh City
Bài gởi: 2,413
Thanks: 2,165
Thanked 4,188 Times in 1,381 Posts
Gửi tin nhắn qua Yahoo chát tới huynhcongbang
Trích:
Nguyên văn bởi nvthanh1994 View Post
Bài 1
Trên mặt phẳng tọa độ có một con cào cào ở điểm $(1;1) $.Nó có thể nhảy từ A sang B khi tam giác OAB có diện tích bằng $\dfrac{1}{2} $
1. Tìm các điểm $(m,n) $ sao cho con cào cào có thể nhảy đến đó sau hữu hạn bước
2.CMR con cào cào có thể nhảy đến $(m,n) $ kể trên sau ít hơn $|m-n| $ bước
[/TEX]
Bài này có lẽ liên quan đến một bài dãy số.
Gọi $A(x_1,y_1), B(x_2,y_2) $ là tọa độ của các điểm mà con cào cào có thể nhảy qua.
Diện tích tam giác OAB chính là:
$S_{OAB} = \frac{1}{2}|x_1y_2-x_2y_1| $.
Do đó, các điểm trong đề bài thuộc dãy số xác định như sau:
$x_0=y_0=1,x_{n+1}y_n-x_ny_{n+1} = \pm 1, n=0, 1,2,... $.
Đến đây đưa bài toán về tìm điều kiện của m và n sao cho $(m,n) $ thuộc dãy số trên.
[RIGHT][I][B]Nguồn: MathScope.ORG[/B][/I][/RIGHT]
 
huynhcongbang is offline   Trả Lời Với Trích Dẫn
Old 09-04-2011, 10:08 PM   #7
Traum
Moderator
 
Traum's Avatar
 
Tham gia ngày: Nov 2007
Đến từ: cyber world
Bài gởi: 413
Thanks: 14
Thanked 466 Times in 171 Posts
Bài 1:

Nhận xét 1:Giả sử tại một thời điểm, con cào cào ở đỉnh $A = (a,b) $ thì nó nhảy sang được đỉnh $B = (c,d) $ khi và chỉ khi $|ad-bc| = 1 $.

Thật vậy diện tích của tam giác $AOB $ là $\frac{1}{2}|ad-bc| $, nên con cào cào có thể nhảy từ đỉnh $A $ sang $B $ khi và chỉ khi $|ad-bc|=1 $. Nhận xét 1 được chứng minh.

Nhận xét 2: Với $(m,n) $ là cặp nguyên tố cùng nhau thì tồn tại a và b nguyên tố cùng nhau sao cho $|mb-na| = 1 $ và $|a-b|\le |m-n|-1 $

Chứng minh: không mất tổng quát, giả sử $m>n $, hiển nhiên tồn tại $1\le b\le n-1 $ sao cho $mb-1 $ chia hết cho $ n $
đặt $ a = (mb-1)/n $ ta có $ a<m $ và $ |mb-na| = 1 $. Hơn nữa $ mb-na = 1 $ nên $ |n(b-a)| = |(m-n)b-1| < |(m-n)n| $ ( do $ b<n $), suy ra $ |b-a|\le |m-n|-1 $. Nhận xét 2 được chứng minh.

Từ $ 2 $ nhận xét trên với điều kiện vị trí ban đầu của con cào cào là $ (1,1) $ ta có
1. Các cặp thỏa mãn là $(m,n) $ nguyên tố cùng nhau
2. Con cào cào có thể nhảy đến sau không quá $|m-n| $ bước.
[RIGHT][I][B]Nguồn: MathScope.ORG[/B][/I][/RIGHT]
 
__________________
Traum is giấc mơ.

thay đổi nội dung bởi: Traum, 10-04-2011 lúc 12:28 AM
Traum is offline   Trả Lời Với Trích Dẫn
The Following 5 Users Say Thank You to Traum For This Useful Post:
buikhacduong (09-04-2011), huynhcongbang (09-04-2011), Lan Phuog (13-04-2011), n.v.thanh (09-04-2011), toannh (09-04-2011)
Old 09-04-2011, 10:17 PM   #8
Traum
Moderator
 
Traum's Avatar
 
Tham gia ngày: Nov 2007
Đến từ: cyber world
Bài gởi: 413
Thanks: 14
Thanked 466 Times in 171 Posts
Bài 3: Max = $\sqrt{2(n-1)(n-2)} $.

Chứng minh: $n(n-1) = x_1^2 + ...+x_n^2 \ge x_1^2+x_2^2 + \frac{n-2}(x_3+...+x_n)^2 = x_1^2 + x_2^2 + \frac{1}{n-2}(x_1+x_2)^2 \ge \frac{1}{2}(x_1+x_2)^2 + \frac{1}{n-2}(x_1+x_2)^2 = \frac{n}{2(n-2)}(x_1+x_2)^2. $ Do đó $(x_1+x_2)^2\le 2(n-1)(n-2) $.
Dấu đẳng thức xảy ra chẳng hạn $x_1 = x_2 = \sqrt{\frac{(n-1)(n-2)}{2}}, $ $x_3 = x_3 =...=x_n = -\sqrt{\frac{2(n-1)}{n-2}} $


Phần min có lẽ là $= 2 $ nếu $n\ge 4 $ ví dụ với $x_1=...=x_{n-1} = 1, x_{n}=-n+1 $.
Với $n = 3 $ thì min = $1 $ ví dụ $x_1 = 2,x_2=x_3 = -1 $
[RIGHT][I][B]Nguồn: MathScope.ORG[/B][/I][/RIGHT]
 
__________________
Traum is giấc mơ.

thay đổi nội dung bởi: Traum, 10-04-2011 lúc 03:27 PM
Traum is offline   Trả Lời Với Trích Dẫn
The Following User Says Thank You to Traum For This Useful Post:
buikhacduong (09-04-2011)
Old 10-04-2011, 12:00 AM   #9
hien123
+Thành Viên+
 
Tham gia ngày: Sep 2010
Đến từ: THPT chuyên Phan Bội Châu, Nghệ An
Bài gởi: 353
Thanks: 19
Thanked 261 Times in 165 Posts
Trích:
Nguyên văn bởi huynhcongbang View Post
Mình ủng hộ bài 2 trước:
Trước hết, ta chứng minh bổ đề sau:
* Cho tam giác ABC ngoại tiếp (I) có tiếp điểm của (I) lên AB, AC lần lượt là E, F. Đường thẳng qua B, song song với AC cắt EF tại K; CK cắt AB tại G. Chứng minh rằng tam giác AGI vuông tại I.

Chứng minh:
Do BK // AC nên tam giác BKF cân tại B, suy ra: $BK=BF = p-b $.



Theo định lí Thales thì:
$\frac{BG}{AG}=\frac{BK}{AC}=\frac{p-b}{b} \Rightarrow \frac{AB}{AG} = \frac{p}{b}\Rightarrow AG = \frac{bc}{p} $
Mà $AF=p-a $ nên $\frac{AF}{AG}=\frac{p(p-a)}{bc} $.
Ta cũng có: $AI = \frac{AF}{\sin \frac{A}{2}}, AH = AF. \sin \frac{A}{2} $.
Do đó: $\frac{AH}{AI}=\sin^2 \frac{A}{2} = \frac{1-\cos A}{2} = \frac{1-\dfrac{b^2+c^2-a^2}{2bc}}{2} = \frac{p(p-a)}{bc} $
Suy ra: $\frac{AF}{AG} =\frac{AH}{AI} $.
Tức là AGI vuông tại I.
Bổ đề được chứng minh.

Trở lại bài toán:



1/ Gọi M, N lần lượt là giao điểm của QE với AB và PF với AC.
Theo bổ đề trên, ta thấy rằng tam giác OMA và ONA lần lượt vuông tại O nên các điểm M, N cố định.
2/ Đặt $AB=AC=a, BP=x, CQ=y $. Chu vi của tam giác APQ là $2(a+x+y) $.
Theo bổ đề trên, ta tính được:
$PM =AP - \frac{2AP.PQ}{AP+AQ+PQ}=\frac{(a+x)x}{a+x+y} $ và
$QN =AQ - \frac{2AQ.PQ}{AP+AQ+PQ}=\frac{(a+y)y}{a+x+y} $.
Ta sẽ chứng minh rằng $\frac{xy(a+x)(a+y)}{(a+x+y)^2} $ không đổi.
Thật vậy:
Diện tích của tam giác APQ cùng bằng:
$R(AP+AQ+PQ) = \sin \widehat{BAC}.AP.AQ \Leftrightarrow \frac{(a+x)(a+y)}{a+x+y}=\frac{R}{ \sin \widehat{BAC}} $.
Tức là tỉ số: $\frac{(a+x)(a+y)}{a+x+y} = k $ không đổi, với $k=\frac{R}{ \sin \widehat{BAC}} $
Từ $(a+x)(a+y)=k(a+x+y) \Leftrightarrow a(a+x+y)+xy = k(a+x+y) \Leftrightarrow a+ \frac{xy}{a+x+y} = k $, suy ra tỉ số $\frac{xy}{a+x+y} $ cũng không đổi.
Ta có đpcm.
Một cách khác:
[RIGHT][I][B]Nguồn: MathScope.ORG[/B][/I][/RIGHT]
 
File Kèm Theo
Kiểu File : doc Bài2VMO2011.doc (33.0 KB, 174 lần tải)

thay đổi nội dung bởi: hien123, 10-04-2011 lúc 08:40 AM
hien123 is offline   Trả Lời Với Trích Dẫn
The Following 2 Users Say Thank You to hien123 For This Useful Post:
cattuong (30-05-2011), hoanghai_vovn (10-04-2011)
Old 10-04-2011, 12:15 AM   #10
kien10a1
+Thành Viên+
 
kien10a1's Avatar
 
Tham gia ngày: Feb 2011
Đến từ: Vĩnh Yên- Vĩnh Phúc
Bài gởi: 371
Thanks: 43
Thanked 263 Times in 153 Posts
Gửi tin nhắn qua Yahoo chát tới kien10a1
Nếu vậy thì cho em hỏi kết luận của phần a bài 1 là gì ạ?
[RIGHT][I][B]Nguồn: MathScope.ORG[/B][/I][/RIGHT]
 
kien10a1 is offline   Trả Lời Với Trích Dẫn
Old 10-04-2011, 12:26 AM   #11
Traum
Moderator
 
Traum's Avatar
 
Tham gia ngày: Nov 2007
Đến từ: cyber world
Bài gởi: 413
Thanks: 14
Thanked 466 Times in 171 Posts
Trích:
Nguyên văn bởi kien10a1 View Post
Nếu vậy thì cho em hỏi kết luận của phần a bài 1 là gì ạ?
Quên mất, đáp số là tất cả các cặp $(m,n) $ nguyên tố cùng nhau.
[RIGHT][I][B]Nguồn: MathScope.ORG[/B][/I][/RIGHT]
 
__________________
Traum is giấc mơ.
Traum is offline   Trả Lời Với Trích Dẫn
Old 10-04-2011, 02:39 AM   #12
hoanghai_vovn
+Thành Viên+
 
hoanghai_vovn's Avatar
 
Tham gia ngày: Jul 2010
Đến từ: Asia
Bài gởi: 208
Thanks: 303
Thanked 111 Times in 64 Posts
Trích:
Nguyên văn bởi hien123 View Post
Một cách khác:
Bạn có thể giải thích cho mình tại sao ở câu a, hai tam giác PMO và ONQ đồng dạng với nhau không? Mình nghĩ nếu chứng minh được điều này thì có ngay M, O, N thẳng hàng và MN // BC rồi còn gì?
[RIGHT][I][B]Nguồn: MathScope.ORG[/B][/I][/RIGHT]
 
__________________
Hate me first, love me later!
hoanghai_vovn is offline   Trả Lời Với Trích Dẫn
Old 10-04-2011, 08:41 AM   #13
hien123
+Thành Viên+
 
Tham gia ngày: Sep 2010
Đến từ: THPT chuyên Phan Bội Châu, Nghệ An
Bài gởi: 353
Thanks: 19
Thanked 261 Times in 165 Posts
Trích:
Nguyên văn bởi hoanghai_vovn View Post
Bạn có thể giải thích cho mình tại sao ở câu a, hai tam giác PMO và ONQ đồng dạng với nhau không? Mình nghĩ nếu chứng minh được điều này thì có ngay M, O, N thẳng hàng và MN // BC rồi còn gì?
Mình sửa lại rồi đó.
[RIGHT][I][B]Nguồn: MathScope.ORG[/B][/I][/RIGHT]
 
hien123 is offline   Trả Lời Với Trích Dẫn
Old 10-04-2011, 08:44 AM   #14
can_hang2008
+Thành Viên+
 
Tham gia ngày: Mar 2009
Bài gởi: 310
Thanks: 5
Thanked 751 Times in 187 Posts
Trích:
Nguyên văn bởi nvthanh1994 View Post
Nhìn qua thấy chắc bài 3 khó nhất.
Bài 3 không khó như em nghĩ đâu. Thật ra dạng bài này đã từng xuất hiện rồi, chỉ khác một chút thôi.

Nhưng bài này anh thấy rất hay vì nó "kill" được thói quen "học dạng" của nhiều bạn.
[RIGHT][I][B]Nguồn: MathScope.ORG[/B][/I][/RIGHT]
 
__________________
The love makes us stronger!

Võ Quốc Bá Cẩn

thay đổi nội dung bởi: can_hang2008, 10-04-2011 lúc 09:20 AM
can_hang2008 is offline   Trả Lời Với Trích Dẫn
Old 10-04-2011, 10:14 AM   #15
nbkschool
+Thành Viên+
 
nbkschool's Avatar
 
Tham gia ngày: Dec 2007
Đến từ: SMU Residence @Prinsep Hostel, 83 Prinsep Street, Singapore
Bài gởi: 400
Thanks: 72
Thanked 223 Times in 106 Posts
Đề câu 1 có thiếu không nhỉ?Các điểm nó nhảy qua buộc phải có tọa độ nguyên hay thế nào?Nếu không cần thì chỉ cần 3 bước là max thôi.
[RIGHT][I][B]Nguồn: MathScope.ORG[/B][/I][/RIGHT]
 
__________________
"Apres moi,le deluge"
nbkschool is offline   Trả Lời Với Trích Dẫn
The Following User Says Thank You to nbkschool For This Useful Post:
thichtoanhoc (19-04-2011)
Trả lời Gởi Ðề Tài Mới

Bookmarks

Ðiều Chỉnh
Xếp Bài

Quuyền Hạn Của Bạn
You may not post new threads
You may not post replies
You may not post attachments
You may not edit your posts

BB code is Mở
Smilies đang Mở
[IMG] đang Mở
HTML đang Tắt

Chuyển đến


Múi giờ GMT. Hiện tại là 01:08 PM.


Powered by: vBulletin Copyright ©2000-2024, Jelsoft Enterprises Ltd.
Inactive Reminders By mathscope.org
[page compression: 120.40 k/137.34 k (12.34%)]